You are on page 1of 24

Chapter

p 12:
Displacement Method of Analysis: Moment Distribution

Structural Analysis 7th Edition in SI Units


Russell C. Hibbeler

General Principles & Definition

• Moment distribution is a method of successive


approximations
pp that mayy be carried out to anyy
desired degree of accuracy
• The method begins by assuming each joint of a
structure is fixed
• By unlocking and locking each joint in succession,
the internal moments at the joints are “distributed
distributed””
& balanced until the joints have rotated to their
final or nearly final positions

1
General Principles & Definition

• Member stiffness factor


4 EI
K
L
• Joint stiffness factor
• The total stiffness factor of joint A is

K T   K  4000  5000  1000  10000

General Principles & Definition

• Distribution Factor (DF)


• That
at fraction
act o oof the
t e tota
total resisting
es st g moment
o e t supp
supplied
ed
by the member is called the distribution factor (DF)

Mi K i
DFi  
M   Ki
K
DF 
K

2
General Principles & Definition

• Member relative stiffness factor


• Qu
Quite
te often
o te a continuous
co t uous bea
beam o
or a frame
a e will be
made from the same material
• E will therefore be constant
I
KR 
L

General Principles & Definition

• Carry
Carry--over (CO) factor
 4 EI   2 EI 
M AB    A ; M BA    A
 L   L 
• Solving for  and equating these eqn,
M BA  0.5M AB
• The moment M at the pin induces a moment of M’ M’
= 0.5M
0 5M at the wall
• In the case of a beam with the far end fixed, the
CO factor is +0.5

3
General Principles & Definition

• Carry
Carry--over (CO) factor
• The
e plus
p us sign
s g indicates
d cates both
bot moments
o e ts act in tthe
e
same direction
• Consider the beam K BA 
4 E (120)(10 6 )
 4 E ( 40)(10 6 )mm 4 / m
3
4 E (240)(10 6 )
K BC   4 E (60)(10 6 ) mm 4 / m
4
4 E (40)
DFBA   0. 4
4 E ( 40)  4 E (60)
4 E (60)
DFBC   0. 6
4 E ( 40)  4 E (60)

General Principles & Definition

4 E (40)
DFAB  0
  4 E (40)
4 E (60)
DFCB  0
  4 E (60)

• Note that the above results could also have been


obtained if the relative stiffness factor is used
wL2
( FEM ) BC    8000kNm
12
wL2
( FEM ) CB   8000kNm
12

4
General Principles & Definition

• We begin by assuming joint B is fixed or locked


• The fixed end moment at B then holds span BC in
this fixed or locked position
• To correct this, we will apply an equal but opposite
moment of 8000Nm to the joint and allow the joint
to rotate freely

General Principles & Definition

• As a result, portions of this moment are distributed


in spans
p BC and BA in accordance with the DFs of
these spans at the joint
• Moment in BA is 0.4(8000) = 3200Nm
• Moment in BC is 0.6(8000) = 4800Nm
• These moment must be carried over since
moments a aree de
developed
eloped at the fa
far ends of the
span

5
General Principles & Definition

• Using the carry


carry--over factor of +0.5, the results are
shown
• The steps are usually presented in tabular form
• CO indicates a line where moments are distributed
then carried over
• In this particular case only one cycle of moment
dist ib tion is necessary
distribution necessa
• The wall supports at A and C “absorb
absorb”” the
moments and no further joints have to be
balanced to satisfy joint equilibrium

General Principles & Definition

6
Example 12.2

Determine the internal moment at each support of the beam. The


moment of inertia of each span is indicated.

Solution

A moment does not get distributed in the overhanging span AB


So the distribution factor (DF)BA =0
Span BC is based on 4EI/L since the pin rocker is not at the far
end of the beam

4 E (300)(10 6 )
K BC   300(10 6 ) E
4
4 E (240)(10 6 )
K CD   320(10 6 ) E
3

7
Solution

DFBC  1  ( DF ) BA  1  0  1

300 E
DFCB   0.484
300 E  320 E
DFCD  0.516; DFDC  0

Due to overhang, ( FEM ) BA  2000 N (2m)  4000 Nm

wL2
( FEM ) BC    2000 Nm
12
wL2
( FEM ) CB   2000 Nm
12

Solution

•The overhanging span requires the internal moment to the left of


B to be +4000Nm.
•Balancing
B l i att joint
j i t B requires
i an internal
i t l momentt off –4000Nm
4000N tto
the right of B.
•-2000Nm is added to BC in order to satisfy this condition.
•The distribution & CO operations proceed in the usual manner.
Since the internal moments are known, the moment diagram for
the beam can be constructed.

8
Solution

Stiffness--Factor Modifications
Stiffness

• The previous e.g. of moment distribution, we have


considered each beam span p to be constrained byy a
fixed support at its far end when distributing &
carrying over the moments
• In some cases, it is possible to modify the stiffness
factor of a particular beam span & thereby simplify
the process of moment distribution

9
Stiffness--Factor Modifications
Stiffness

• Member pin supported at far end


• Ass sshown
o tthe
e app
applied
ed moment
o e t M rotates
otates e
end
d A by
an amt 
• To determine , the shear in the conjugate beam at
A’ must be determined

1 M  2 
 M B'  0 V ' A ( L )    L L   0
2  EI   3 
3L 3EI
V 'A    M  
EI L

Stiffness--Factor Modifications
Stiffness

• Member pin supported at far end (cont’


(cont’d)
• The
e st
stiffness
ess factor
acto in tthe
e bea
beam iss
3EI
K
L
• The CO factor is zero, since the pin at B does not
support a moment
• By comparison,
comparison if the far end was fixed supported,
supported
the stiffness factor would have to be modified by ¾
to model the case of having the far end pin
supported

10
Stiffness--Factor Modifications
Stiffness

• Symmetric beam & loading


• Thee be
bending
bending-
d g-moment
o e td diagram
ag a for o tthe
e bea
beam will
also be symmetric
• To develop the appropriate stiffness-
stiffness-factor
modification consider the beam
• Due to symmetry, the internal
moment at B & C are equal
• Assuming this value to
be M, the conjugate
beam for span BC is shown

Stiffness--Factor Modifications
Stiffness

• Symmetric beam & loading (cont’


(cont’d)
 M  L
 M C '  0 - V ' B ( L )    L   0
 EI   2 
ML 2 EI
V 'B    M  
2 EI L
2 EI
K 
L
• Moments for only half the beam can be distributed
provided the stiffness factor for the center span is
computed

11
Stiffness--Factor Modifications
Stiffness

• Symmetric beam with asymmetric loading


• Co
Consider
s de tthe
e bea
beam as sshown
o
• The conjugate beam for its center span BC is shown
• Due to its asymmetric loading, the internal moment
at B is equal but opposite to that at C

Stiffness--Factor Modifications
Stiffness

• Symmetric beam with asymmetric loading


• Assuming p  at each
g this value to be M,, the slope
end is determined as follows:
 M C'  0
 1  M  L  5 L  1  M  L  L 
- V ' B ( L)             0
 2  EI  2  6  2  EI  2  6 
ML 6 EI
V 'B    M  
6 EI L
6 EI
K
L

12
Example 12.4

Determine the internal moments at the supports of the beam


shown below. The moment of inertia of the two spans is shown in
the figure
figure.

Solution

•The beam is roller supported at its far end C.


•The stiffness of span BC will be computed on the basis of K =
3EI/L
•We have:

4 EI 4 E (120)(10 6 )
K AB    160(10 6 ) E
L 3
3EI 3E (240)(10 6 )
K BC    180(10 6 ) E
L 4

13
Solution

160 E
DFAB  0
  160 E
160 E
DFBA   0.4706
160 E  180 E
180 E
DFBC   0.5294
160 E  180 E
180 E
DFCB  1
180 E
wL2  6000(4) 2
( FEM ) BC     12000 Nm
8 8

Solution

The forgoing data are entered into table as shown.


The moment distribution is carried out.
By comparison, the method considerably simplifies the
distribution.
The beam’s end shears & moment diagrams are shown.

14
Moment distribution for frames:
No sidesway
• Application of the moment
moment--distribution method for
frames havingg no sideswayy follows the same
procedure as that given for beam

Example 12.5

Determine the internal moments at the joints of the frame as


shown. There is a pin at E and D and a fixed support at A. EI is
constant
constant.

15
Solution

By inspection, the pin at E will prevent the frame will sidesway.


The stiffness factors of CD and CE can be computed using K =
3EI/L since
i far
f endsd are pinned.
i d
The 60kN load does not contribute a FEM since it is applied at
joint B.
4 EI 4 EI 3EI 3EI
K AB  ; K BC  ; K CD  ; K CE 
5 6 5 4
DFAB  0

4 EI / 5
DFBA   0.545
4 EI / 5  4 EI / 6
DFBC  1  0.545  0.455

Solution

4 EI / 6
DFCB   0.330
4 EI / 6  3EI / 5  3EI / 4
3EI / 5
DFCD   0.298
4 EI / 6  3EI / 5  3EI / 4
DFCE  1  0.330  0.298  0.372

DFDC  1; DFEC  1

 wL2
( FEM ) BC   135kNm
12
wL2
( FEM ) CB   135kNm
12

16
Solution

•The data are shown in table.


•The distribution of moments successively goes to joints B & C.
•The final moment are shown on the last line.
•Using these data, the moment diagram for the frame is
constructed as shown.

Moment distribution for frames: Sidesway

• To determine sidesway and the internal moments


at the jjoints using
g moment distribution,, we will use
the principle of superposition
• The frame shown is first held from sidesway by
applying an artificial joint support at C
• Moment distribution is applied & by statics, the
restraining force R is determined

17
Moment distribution for frames: Sidesway

• The equal but opposite restraining force is then


applied
pp to the frame The moments in the frame
are calculated

Moment distribution for frames: Sidesway

• Multistory frames
• Multistory
u t sto y frameworks
a e o s may
ay have
a e seseveral
ea
independent joints disp
• Consequently, the moment distribution analysis
using the above techniques will involve more
computation

18
Moment distribution for frames: Sidesway

• Multistory frames
• Thee st
structure
uctu e sshown
o ca
can have
a e 2 independent
depe de t jo
jointt
disp since the sidesway of the first story is
independent of any disp of the second story

Moment distribution for frames: Sidesway

• Multistory frames
• These
ese d
disp
sp are
a e not
ot known
o initially
ta y
• The analysis must proceed on the basis of
superposition
• 2 restraining forces R1 and R2 are applied
• The fixed end moments are determined &
distributed
• Using the eqn of eqm, the numerical values of R1
and R2 are then determined

19
Moment distribution for frames: Sidesway

• Multistory frames
• Thee restraint
est a t at tthe
e floor
oo oof tthe
e first
st sto
story
y iss
removed & the floor is given a disp
• This disp causes fixed end moment (FEMs) in the
frame which can be assigned specific numerical
values
• By distributing these moments & using the eqn of
eqm, the associated numerical values of R1’ and R2’
can be determined

Moment distribution for frames: Sidesway

• Multistory frames
• In a similar
s a manner,
a e , the
t e floor
oo of
o the
t e seco
second
d sto
story
y iss
then given a disp
• With reference to the restraining forces we require
equal but opposite application of R1 and R2 to the
frame such that:

R2  C ' R ' 2 C ' R"2

R1  C ' R'1 C ' R"1

20
Moment distribution for frames: Sidesway

• Multistory frames
•SSimultaneous
u ta eous sosolution
ut o oof tthese
ese eq
eqn yyields
e ds tthe
e
values of C’
C’ and C”
C”
• These correction factors are then multiplied by the
internal joint moments found from moment
distribution
• The resultant moments are found by adding these
corrected moments to those obtained for the frame

Example 12.6

Determine the moments at each joint of the frame shown. EI is


constant.

21
Solution

First, we consider the frame held from sidesway


16(4) 2 (1)
( FEM ) BC    10.24kNm
kN
(5) 2

16(1) 2 (4)
( FEM ) CB   2.56kNm
(5) 2

The stiffness factor of each span is computed on the basis of


4EI/L or using relative stiffness factor I/L

Solution

The DFs and the moment distribution are shown in the table.
The eqn of eqm are applied to the free body diagrams of the
columns
l in
i order
d to
t determine
d t i Ax and d Dx
From the free body diagram of the entire frame, the joint restraint
R has a magnitude of
 Fx  0; R  1.73kN  0.81kN  0.92kN

22
Solution

•An equal but opposite value of R = 0.92kN must be applied to


the frame at C and the internal moments computed.
•We
W assume a force
f R’ is
i applied
li d att C causing
i the
th frame
f to
t deflect
d fl t
as shown.
•The joints at B and C are temporarily restrained from rotating.
•As a result, the FEM at the ends of the columns are determined.

Solution

•Since both B and C happen to be displaced the same amount and


AB and DC have the same E, I and L, the FEM in AB will be the
same as that in DC.
DC
•As shown we will arbitrarily assumed this FEM to be
( FEM ) AB  ( FEM ) BA  ( FEM ) CD  ( FEM ) DC  100kNm

The moment distribution of the FEM is shown below.

23
Solution

•From the eqm, the horizontal reactions at A and D are calculated.


•For the entire frame, we require:

 Fx  0; R '  28  28  56kN
•R’=56kN creates the moments tabulated below
•Corresponding moments caused by R = 0.92kN can be
determined by proportion
M AB  2.88 
0.92
 80  1.57kNm
56.0
M BA  4.79kNm; M BC  4.79kNm; M CB  3.71kNm

M CD  3.71kNm; M DC  2.63kNm

24

You might also like